Helppp me please I need to graduate

Helppp Me Please I Need To Graduate

Answers

Answer 1

(5x³-7)(2x²+1)

OPTION D is the correct answer

Helppp Me Please I Need To Graduate

Related Questions

`find the equatifindon of tangent and normal of parabola y²=16ax at point whose coordinate is -5a. ​

Answers

Answer:

[tex] \rm \displaystyle y _{ \rm tangent} = - \frac{8}{5} x - \frac{5}{2} a[/tex]

[tex] \rm \displaystyle y _{ \rm normal} = \frac{5}{8} x - \frac{765}{128} a[/tex]

Step-by-step explanation:

we are given a equation of parabola and we want to find the equation of tangent and normal lines of the Parabola

finding the tangent line

equation of a line given by:

[tex] \displaystyle y = mx + b[/tex]

where:

m is the slopeb is the y-intercept

to find m take derivative In both sides of the equation of parabola

[tex] \displaystyle \frac{d}{dx} {y}^{2} = \frac{d}{dx} 16ax [/tex]

[tex] \displaystyle 2y\frac{dy}{dx}= 16a[/tex]

divide both sides by 2y:

[tex] \displaystyle \frac{dy}{dx}= \frac{16a}{2y}[/tex]

substitute the given value of y:

[tex] \displaystyle \frac{dy}{dx}= \frac{16a}{2( - 5a)}[/tex]

simplify:

[tex] \displaystyle \frac{dy}{dx}= - \frac{8}{5}[/tex]

therefore

[tex] \displaystyle m_{ \rm tangent} = - \frac{8}{5}[/tex]

now we need to figure out the x coordinate to do so we can use the Parabola equation

[tex] \displaystyle ( - 5a {)}^{2} = 16ax [/tex]

simplify:

[tex] \displaystyle x = \frac{25}{16} a[/tex]

we'll use point-slope form of linear equation to get the equation and to get so substitute what we got

[tex] \rm \displaystyle y - ( - 5a)= - \frac{8}{5} (x - \frac{25}{16} a)[/tex]

simplify which yields:

[tex] \rm \displaystyle y = - \frac{8}{5} x - \frac{5}{2} a[/tex]

finding the equation of the normal line

normal line has negative reciprocal slope of tangent line therefore

[tex] \displaystyle m_{ \rm normal} = \frac{5}{8}[/tex]

once again we'll use point-slope form of linear equation to get the equation and to get so substitute what we got

[tex] \rm \displaystyle y - ( - 5a)= \frac{5}{8} (x - \frac{25}{16} a)[/tex]

simplify which yields:

[tex] \rm \displaystyle y = \frac{5}{8} x - \frac{765}{128} a[/tex]

and we're done!

( please note that "a" can't be specified and for any value of "a" the equations fulfill the conditions)

Answer:

In attachment

Step-by-step explanation:

For answer refer to attachment .

What is the volume of a sphere with a diameter of 57.1 cm, rounded to the nearest tenth of a cubic centimeter?

Answers

Answer:

so we have to use the formula 4/3PiR^3

which if we do we get the volume of

V≈97478.08

Answer:

V≈97478.08cm³

Step-by-step explanation:

Using the formulas

V=4/3πr3

d=2r

V=1/6πd3

D=1

1/6·πd3 1/6π ·57.13≈97478.07565cm³

I hope this helps. I worked hard on this one.

the students in charge of the class booth at a carnival would like to earn $3 for every item they sell. they spent $55 for the materials to make the items. solve the inequality 3x-55_>65 which represents how many items they need to sell to make profit of at least $65

Answers

Answer:

x ≥ 40

Step-by-step explanation:

3x - 55 ≥ 65

combine like terms

3x  ≥ 65 + 55

3x  ≥120

divide both sides of the equation by 3

x  ≥ 40

The number of points Marcus scored in different rounds of a computer board
game is shown 236, 342, 328, 352, 352, 306, 317
What score does Marcus need in his next game to have a mean of exactly 325?

HELP FAST PLZ

Answers

Answer:583

Step-by-step explanation:

I took the test

According to the general equation for conditional probability, if P(An B9 =
001 -
and P(B)
=
7
18
what is P(
AB)?
A. 5/7
B. 3/7
C. 4/7
D. 2/7

Answers

P is answer A .5/7 for ur question

A large soda bottle cap has a radius of 1+2 /2 centimeters. What is the area of the bottle cap? (R the area of a circle is no where r is the radius). 1 + 2v2 A. (5-12 - square IT square centimeters​

Answers

Answer:

bottle top

Step-by-step explanation:

Find the value of X (in this picture)​

Answers

Answer:

x = 89

Step-by-step explanation:

The sum of the angles of a triangle add to 180 degrees

x+32+ 59 = 180

Combine like terms

x + 91 = 180

Subtract 91 from each side

x +91-91= 180-91

x = 89

Select the correct answer. Which is the simplified form of the expression ? A. B. C. D.

Answers

Answer:

where is the question

Step-by-step explanation:

send question

Answer:

The answer is B: 67/10x + 9

Step-by-step explanation:

I did the test

PLEASE HELP!!!

WILL MARK BRAINLIEST!!!

Solve for X.
Multiple choice!

Thank you!

Answers

Answer:

9

Step-by-step explanation:

the 2 angles create a right angle ( indicated by the little square.)

right angles have a measure of 90 degrees

hence, 90 = 72 + 2x

( note that we just created an equation that we can use to solve for x )

we now solve for x using the equation we created.

90 = 72 + 2x

step 1 subtract 72 from each side

90 - 72 = 18

72 - 72 cancels out

we now have 18 = 2x

step 2 divide each side by 2

18 / 2 = 9

2x / 2 = x

we're left with x = 9

Kind of struggling with this would anyone be able to explain step by step? Id really appreciate it :)

Answers

Answer:

75

Step-by-step explanation:

this problem statement tries to confuse us. there is no pi involved - also because there are no round surfaces and edges involved.

what do you think the surface area of such a 4-sided pyramid consists of ?

think !

there are the 4 walls or sides. and then there is the bottom too (if you lift the pyramid up, yes, the "floor" or bottom also belongs to the surface area).

each "wall" or side is actually a triangle.

and the bottom is a square.

what is the area of a triangle (At) ?

At = base length × height / 2 = 3×11/2 = 33/2 in²

what is the area of a square (As) ?

it is a special form of a rectangle, where length = width.

the area of a rectangle (Ar) = length × width.

and the area of a square (As), where length = width is

As = side length × side length = length² = 3² = 9 in²

our pyramid now has 4 "walls", all of the same size, so we need 4 times the area of such a single triangle.

and then add the bottom area.

=> the pyramid surface area (Ap) is

Ap = 4 × At + As = 4×33/2 + 9 = 132/2 + 9 = 66+9 = 75 in²

Can someone please help me out? It would really make me happier if you did.
The difference between two numbers is 16. The first number is three times the other number. What are the numbers?
A. 8 and 25
B. 8 and 24
C. 9 and 24
D. 9 and 25
No links or fake answers pls. Thank you for understanding.

Answers

Answer:

B) 8 and 24.

Step-by-step explanation:

Just by looking at the answer choices, since one number must be three times the other, we can determine that the correct answer is B.

Regardless, let's work this out mathematically. Let the first number be a and the second number be b.

Their difference is 16. Hence:

[tex]a-b=16[/tex]

The first number is three times the second number. So:

[tex]a=3b[/tex]

Substitute:

[tex]3b-b=16[/tex]

Solve for b. Subtract:

[tex]2b=16[/tex]

And divide both sides by two:

[tex]b=8[/tex]

So, the second number is 8.

And since the first number is three times the second, the first number is 24.

Our answer is B as expected.

In the figure, the circle with center A transforms to produce the circle with center A'Which transformation took place?

A. dilation about the origin by factor of 3
B. vertical stretch with respect of the x-axis by a factor of 2
C. Horizontal stretch with respect to y-axis by factor.
D. dilation about the origin by a factor of 2​

Answers

Answer:

D

Step-by-step explanation:

If you look at the centerand radius of a circle (those are the two most important things when defining a circle), you will see, they shrank by a factor of two, so since it affected both x and y, it had to be from the origin.

Grandmother decides to give a monthly allowance to her granddaughter from the trust fund. The allowance is to start at $ at the birth of her granddaughter, and then increase by $ each month afterwards. Describe the grandmother's gifts, approximately, in terms of a continuous rate (in dollars per year). (Use a linearly increasing function with no constant term for .)

Answers

Answer:

[tex]y(t) =10t[/tex]

Step-by-step explanation:

The summary of the given parameters are:

[tex]a = 0[/tex] --- initial savings

[tex]m = 10[/tex] --- monthly savings

[tex]t \to[/tex] months

Required

The linear function

The linear function is calculated using:

y(t) = Initial Savings + Monthly Savings * Months

So, we have:

[tex]y(t) =a + m*t[/tex]

[tex]y(t) =0 + 10*t[/tex]

[tex]y(t) =10t[/tex]

► Find the percent change and tell whether it is a percent increase or a
percent decrease.

1 Original amount: 20
End amount: 15

2 Original amount: 30
End amount: 45

3 Original amount: 625
End amount: 550

4 Original amount: 320
End amount: 112

5 Original amount: 165
End amount: 222.75

6 Original amount: 326
End amount: 423.80

7 Original amount: 27
End amount: 38.61

8 Original amount: 60
End amount: 70.02

Answers

Answer:

1 Original amount: 20

End amount: 15

(20 - 15)/20 x 100 = (5/20) x 100 = 25% decrease

2 Original amount: 30

End amount: 45

(45 - 30)/30 x 100 = (15/30) x 100 = 50% increase

3 Original amount: 625

End amount: 550

550/625 =550 ÷ 625 =0.880.88 =0.88 × 100/100 =(0.88 × 100)/100 =88/100 =88% decrease

4 Original amount: 320

End amount: 112

(112-320):320x100 =100 =(112:320-1)x100 =100 =35-100 = -65% decrease

5 Original amount: 165

End amount: 222.75

165/222.75 =165 ÷ 222.75 ≈0.7407407407407410.740740740740741 =0.740740740740741 × 100/100 =(0.740740740740741 × 100)/100 ≈74.074074074074/100 =74.074074074074% ≈74.07% Increase

6 Original amount: 326

End amount: 423.80

(423.8-326):326x100 =100 =(423.8:326-1)x100 =100 =130-100 = 30% Increase

7 Original amount: 27

End amount: 38.61

(38.61-27):27x100 =100 =(38.61:27-1)x100 =100 =143-100 = 43% Increase

8 Original amount: 60

End amount: 70.02

(70.02-60):60x100 =100 =(70.02:60-1)x100 =100 =116.7-100 = 16.7% Increase

need answer !! please

Answers

Step-by-step explanation:

-5(4)+2

-20+2

-18

just put 4 on the place of x

Answer:

[tex]=-18[/tex]

Step-by-step explanation:

[tex]g(x)=-5x+2[/tex]

Let's substitute 4 for x and solve.

[tex]g(4)=-5(4)+2[/tex]

[tex]g(4)=-20+2[/tex]

[tex]g(4)=-18[/tex]

This means that when the function of [tex]g[/tex] is 4, then the [tex]x[/tex] value is [tex]-18[/tex]

Hope this helps.

Cary calculated the surface area of a box in the shape of a rectangular prism. She wrote the equation 148 = 2 (6w + 6h + hw) to represent the width and height of the box. She solved for w and got w = StartFraction 74 minus 6 h Over h + 6 EndFraction Which of the following is an equivalent equation?

Answers

Answer:

the answer is w = 148-12h/12+2h

The equivalent equation is -

[tex]$w=\frac{74-6h}{h+6}[/tex]

We have the equation written by Carly → 148 = 2 (6w + 6h + hw) that represent the width and height of the box in the shape of a rectangular prism.

We have to solve for w.

What do you mean by Equivalent expression ?

Any expression written in a form different from the original form, but gives same result for any input are called equivalent expressions.

Solve for x :  [tex]$log(\frac{x}{\omega}) = 2\pi[/tex]

We have -  

[tex]$log(\frac{x}{\omega}) = 2\pi[/tex]

log(x) - log(ω) = 2π

log(x) = 2π + log(ω)

x = [tex]e^{(2\pi + log(\omega))} = e^{2\pi } \times e^{log(\omega)}[/tex]

According to the question, we have -

148 = 2 (6w + 6h + hw)

(6w + 6h + hw) = 74

hw + 6w = 74  - 6h

w(h + 6) = 74 - 6h

[tex]$w=\frac{74-6h}{h+6}[/tex]

Hence, the equivalent equation is -

[tex]$w=\frac{74-6h}{h+6}[/tex]

To solve more questions on Rearranging expression, visit the link below-

https://brainly.com/question/1824488

#SPJ2

Find the equation of the line that
is perpendicular to y = -4x + 3
and contains the point (8, 1).
y =
x+
Enter

Answers

Answer:

Step-by-step explanation:

L : y = -4x + 3

with a slope of -4.

Any line perpendicular to L will have a slope m such that

m ( -4) = -1, or m = 1/4

Any line perpendicular to L passing through point (a,b) will therefore have the form

P : (y-b) = (x-a)/4

When (a,b) = (8,1) as given, then substitute in P to get

P : (y-1) = (x-8)/4

y=x/4 -2 + 1

y = x/4 -1

I would greatly appreciate it if someone could help me with this. A car travelled 400 km in 5 hours. Joe calculated the speed as 80 km/h but, when he graphed the relation, he calculated a slope of 0.0125. What do you think Joe did incorrectly?​

Answers

It sounds like Joe calculated 5/400 = 0.0125

He divided in the wrong order. It should be 400/5 = 80 since we divide distance over time to get the speed (or rate).

-----------------

In terms of x and y, we basically are saying

slope = rise/run

slope = (change in y)/(change in x)

slope = (change in distance)/(change in time)

slope = (400 km)/(5 hrs)

slope = 80 km per hour is the speed

Which fraction is equal to 3?
0A) 30/10
OB) 1/3
OC) 10/3
OD) 13/1​

Answers

Answer:

The answer is 30/10

Step-by-step explanation:

30 divided by 10 is 3.

Answer: A) 30/10

30/10=3

Explanation: Divide 30 by 10 and solve. You could insert this into your calculator to solve. You should get the answer of 3.

BET YOU CANT SOLVE THIS....

Answers

the perimeter of figure EFGH is 10
Beth can’t man sorry

The center of a circle is at (6,-7) and the diameter of the circle is 22. Which of the following is the equation of the circle? (Part 1 and Part 2)

Answers

Answer:

P1.H.x²+y²=34

centre[h,k]=(0,0)

point=[3,5]

now

radius=[tex]\sqrt{(0-3)²+(0-5)²}=\sqrt{34}[/tex]

now

equation of a circle is;

(x-h)²+(y-k)²=[tex]\sqrt{34}²[/tex]

x²+y²=34

P:2I.(x-6)²+(y+7)²=121

centre[h,k]=(6,-7)

diameter=22

radius[r]=22/2=11

now

equation of a circle is;

(x-h)²+(y-k)²=r²

(x-6)²+(y+7)²=11²

(x-6)²+(y+7)²=121

Add - 3/x + 7y/x .
-4y/2x
-3 + 7y/x
- 10y/x
-3 + 7y/2x

Answers

Answer:

[tex]-\frac{3}{x} + \frac{7y}{x} = \frac{-3+ 7y}{x}[/tex]

Step-by-step explanation:

Given

[tex]-\frac{3}{x} , \frac{7y}{x}[/tex]

Required

Add

The statement can be interpreted as:

[tex]-\frac{3}{x} + \frac{7y}{x}[/tex]

Take LCM

[tex]-\frac{3}{x} + \frac{7y}{x} = \frac{-3+ 7y}{x}[/tex]

10t+[tex]\geq[/tex]130+3.5t

Answers

Answer:

The answer is [tex]t\geq 20[/tex].

Step-by-step explanation:

To solve the inequality, start by solving for the variable [tex]t[/tex].

To solve for the variable [tex]t[/tex], subtract [tex]3.5t[/tex] from both sides. The inequality will look like [tex]6.5t\geq 130[/tex].

Then, divide both sides by 6.5 in order to get the variable [tex]t[/tex] by itself. The inequality answer will look like [tex]t\geq 20[/tex].

Find the volume of the pentagonal prism?

please help me

Answers

Answer:

126 cm^3

Step-by-step explanation:

Use volume formula (Bh=V): 21×6=126

The volume of the pentagonal prism is 126 cm³

What is pentagonal prism?

Pentagonal prism has two pentagonal bases, top and bottom and five rectangular sides.

How to find the the volume of pentagonal?

Volume of the pentagonal prism = Area of the pentagon• height of the prism

Given:

Area of the pentagon = 21 cm²

Height of the prism = 6 cm

The volume of pentagonal prism = Area of the pentagon * Height of the prism

The volume of pentagonal prism = 21 cm² × 6 cm = 126 cm³

Therefore, the volume of pentagonal prism is 126 cm³

Learn more about the volume of pentagonal prism here:

https://brainly.com/question/31028600

#SPJ6

please help zkhdusjdushs​

Answers

Answer:

48

Step-by-step explanation:

16×3=48

therefore the answer is forty eight

i need help with this geometry question pleasee

Answers

Answer:i dont know

Step-by-step explanation:lol

Answer:

tmabm queria saber

Step-by-step explanation:

NEED HELP ASAP
Which equation does the graph of the systems of equations solve?

two linear functions intersecting at 3, negative 2

−one thirdx + 3 = x − 1
one thirdx − 3 = −x + 1
−one thirdx + 3 = −x − 1
one thirdx + 3 = x − 1

Answers

Answer:

[tex]\frac{1}{3}x-3=-x+1[/tex]

Step-by-step explanation:

Since there are plenty of lines that can pass through a single point, he only way to solve this question is by substituting values of [tex]3[/tex] into each equation and seeing if both equations return a value of [tex]-2[/tex].

Starting with the first answer choice:

[tex]-\frac{1}{3}(3)+3=3-1,\\-1+3=3-1,\\2=2[/tex]

Since none of the equations here return a value of [tex]-2[/tex], the correct answer must not include [tex]-\frac{1}{3}x+3[/tex] or [tex]x-1[/tex].

Thus, we can eliminate answer choices A, C, and D, hence the correct answer is [tex]\boxed{\text{B. }\frac{1}{3}x-3=-x+1}[/tex]

equation of the line that has a slope of - 1/2and passes through the points (4,5)

Answers

Answer:

y=-1/2x+7

Step-by-step explanation:

I think its that

Answer:

y = (1/2) x + 3

Step-by-step explanation:

The equation of linear functions is y = m x + b

In this case, we know m = 1/2 , and have point (4,5). We only need to find our y-intercept or b value.

What we can do is substitute what we know into our formula which will give us variable b, or the y-intercept. That will look like:

5 = (1/2) * 4 + b

5 = 2 + b

3 = b

So, the equation is y = (1/2) x + 3 :)

Is 8ft equal to 3yd[tex]8ft=3yd?[/tex]

Answers

Answer:

False

Step-by-step explanation:

There is 3 feet in every yard so if we multiply by 3, we get 9. This is more than 8 so it is NOT equal to 3 yards.

Best of Luck!


Francis borrowed $20 from his dad in the morning. Later, he gave his dad 8
dollars back. What rational number represents the overall amount of money Francis still owes his dad?

Answers

Answer:

13

Step-by-step explanation:

Francis borrowed 20$ and gives his dad 8$ he owes 12$ I hope it helps :)

The answer would be $12 still owed to his father.
Other Questions
x^ 3 + 3 x ^ 2 - 9 x - 27 draw the graph of the first derivative and the second derivative on the same set what is better, a stronger state with a weak national government or weaker states with a strong national government. explain why? why do all states not have bicameral legislature? Given the Vapour Density of a hydrocarbon is 150, what is it's molecular formula. (Trigonometry) Find the values of x and y. Round your answers to the nearest tenth. Summary of how I could use my Video Skills outside of school QUESTION AT THE BOTTOM MAKE SURE YOU KNOW THE ANSWER FIRST PLAESE please helpA circuit contains two light bulbs connected in parallel. What would happen to the brightness of each light bulb if two more light bulbs were added in parallel to the first ones?A.)The brightness of each bulb would decrease because the total resistance of the circuit would decrease.B.)The brightness of each bulb would increase because the total resistance of the circuit would increase.C.)The brightness of each bulb would remain the same even though the total resistance of the circuit would decrease.D.)The brightness of each bulb would remain the same even though the total resistance of the circuit would increase Which of these words could be used to describe a message that combines several different forms of communication?1 autocorrect2 deductible3 multimedia4 subvert What is the impact of social media platforms in protection of human rights What will happen if the condition(red blood cells swelling and bursting), occurs to red blood cells in our body? Completa las siguientes reacciones, nombrando todos los compuestos que intervienen:a) CH2=CH2 + energa b) CH2=CH2 + H2O c) CH2=CH2 + HCl d) CH2=CH2 + Cl2 e) CH2=CH2 + H2 2 Completa las siguientes reacciones, nombrando todos los compuestos que intervienen:a) CH4 + Cl2 b) CH2=CH2 + H2O c) CHCH + H2 d) CH3-COOH + KOH e) CH3OH + CH3-COOH 3 Completa y ajusta la siguientes reacciones nombrando todos los compuestos que intervienen en cada una de ellas:a) CH3-COOH + NaOH b) CH3-CH2I + NH3 c) CH2=CH2 + H2O d) CH3-CH=CH2 + Br2 Part of the budgeting process is summarizing the financial statement effects on the budgeted income statement and the budgeted balance sheet.a. trueb. false PLEASE ANSWER ASAP FOR BRAINLEST!!!!!!!!!!!!!!!!!!!!!!!!!!!! The following are the Pulse Rates of the 12 students in a Math Class76 60 59 60 81 72 91 8080 68 73 77If we create Classes for a relative frequency distribution.For the Class 50-59 , find the frequency A wife suffered from a particularly virulent form of cancer, and had lapsed into a nearly comatose state. Because the doctors had indicated that any treatment they could prescribe would be of little value, her husband decided to administer various poisons to his wife, thinking that they might stimulate her natural body defenses, or kill the cancer cells, resulting in her recovery. He tried doses of many different types of poison. Despite his ministrations, his wife died three days later. An autopsy performed by the county coroner established the cause of death as cancer.If the husband is prosecuted for the murder of his wife, which of these is the best reason why he would be acquitted?A. He was trying to save her life.B. He did not have the necessary malice for his actions to constitute murder.C. Medical science had given her up for dead.D. He did not cause her death. Which statement best describes the significance of the Union victory at Vicksburg to the outcome of the war?The victory gave the Union complete control of the Mississippi River.The victory gave the Union complete control of the Mississippi River.The victory persuaded the French to join the Union cause.The victory persuaded the French to join the Union cause.The victory provided the Union with complete control of the Great Lakes.The victory provided the Union with complete control of the Great Lakes.The victory saw the death of Confederate General Robert E. Lee. no links please helpppppp what does the scale 1:500 000 mean Leon verified that the side lengths 21, 28, 35 form a Pythagorean triple using this procedure. Step 1: Find the greatest common factor of the given lengths: 7 Step 2: Divide the given lengths by the greatest common factor: 3, 4, 5 Step 3: Verify that the lengths found in step 2 form a Pythagorean triple: 3 squared + 4 squared = 9 + 16 = 25 = 5 squared Leon states that 21, 28, 35 is a Pythagorean triple because the lengths found in step 2 form a Pythagorean triple. Which explains whether or not Leon is correct? Yes, multiplying every length of a Pythagorean triple by the same whole number results in a Pythagorean triple. Yes, any set of lengths with a common factor is a Pythagorean triple. No, the lengths of Pythagorean triples cannot have any common factors. No, the given side lengths can form a Pythagorean triple even if the lengths found in step 2 do not. 1. Suly watched a film that was 90 minutes long. 1/6 of the way through the film, thedoorbell rang. The pizza man had arrived with the order.How many minutes of the film had she watched before the pizza man arrived?(3 marks)a.